Question and Answers Forum

All Questions      Topic List

Geometry Questions

Previous in All Question      Next in All Question      

Previous in Geometry      Next in Geometry      

Question Number 206216 by cortano21 last updated on 09/Apr/24

Answered by A5T last updated on 09/Apr/24

HC=(√(108+4))=(√(112))=4(√7)  Let HC meet the circumcircle of the hexagon  at L then, HC×HL=FH×HE=8⇒HL=((2(√7))/7)  sin∠EHC=((√(108))/( (√(112))))=((3(√(21)))/(14))⇒cos∠EHC=((√7)/(14))  ⇒(√(x^2 +2^2 −4x((√7)/(14))))=(√(KD^2 +6^2 −6(√3)×KD))  ⇒x^2 +4−((2x(√7))/7)=KD^2 +36−6(√3)KD...(i)  FK×KD=KC×KL  ⇒((√(108))−KD)KD=(4(√7)−x)(x+((2(√7))/7))  ⇒6(√3)KD−KD^2 =4x(√7)+8−x^2 −((2x(√7))/7)...(ii)  (i)&(ii)⇒32+((2x(√7))/( 7))=4x(√7)+8−((2x(√7))/7)  ⇒x=(√7)≈2.64575

$${HC}=\sqrt{\mathrm{108}+\mathrm{4}}=\sqrt{\mathrm{112}}=\mathrm{4}\sqrt{\mathrm{7}} \\ $$$${Let}\:{HC}\:{meet}\:{the}\:{circumcircle}\:{of}\:{the}\:{hexagon} \\ $$$${at}\:{L}\:{then},\:{HC}×{HL}={FH}×{HE}=\mathrm{8}\Rightarrow{HL}=\frac{\mathrm{2}\sqrt{\mathrm{7}}}{\mathrm{7}} \\ $$$${sin}\angle{EHC}=\frac{\sqrt{\mathrm{108}}}{\:\sqrt{\mathrm{112}}}=\frac{\mathrm{3}\sqrt{\mathrm{21}}}{\mathrm{14}}\Rightarrow{cos}\angle{EHC}=\frac{\sqrt{\mathrm{7}}}{\mathrm{14}} \\ $$$$\Rightarrow\sqrt{{x}^{\mathrm{2}} +\mathrm{2}^{\mathrm{2}} −\mathrm{4}{x}\frac{\sqrt{\mathrm{7}}}{\mathrm{14}}}=\sqrt{{KD}^{\mathrm{2}} +\mathrm{6}^{\mathrm{2}} −\mathrm{6}\sqrt{\mathrm{3}}×{KD}} \\ $$$$\Rightarrow{x}^{\mathrm{2}} +\mathrm{4}−\frac{\mathrm{2}{x}\sqrt{\mathrm{7}}}{\mathrm{7}}={KD}^{\mathrm{2}} +\mathrm{36}−\mathrm{6}\sqrt{\mathrm{3}}{KD}...\left({i}\right) \\ $$$${FK}×{KD}={KC}×{KL} \\ $$$$\Rightarrow\left(\sqrt{\mathrm{108}}−{KD}\right){KD}=\left(\mathrm{4}\sqrt{\mathrm{7}}−{x}\right)\left({x}+\frac{\mathrm{2}\sqrt{\mathrm{7}}}{\mathrm{7}}\right) \\ $$$$\Rightarrow\mathrm{6}\sqrt{\mathrm{3}}{KD}−{KD}^{\mathrm{2}} =\mathrm{4}{x}\sqrt{\mathrm{7}}+\mathrm{8}−{x}^{\mathrm{2}} −\frac{\mathrm{2}{x}\sqrt{\mathrm{7}}}{\mathrm{7}}...\left({ii}\right) \\ $$$$\left({i}\right)\&\left({ii}\right)\Rightarrow\mathrm{32}+\frac{\mathrm{2}{x}\sqrt{\mathrm{7}}}{\:\mathrm{7}}=\mathrm{4}{x}\sqrt{\mathrm{7}}+\mathrm{8}−\frac{\mathrm{2}{x}\sqrt{\mathrm{7}}}{\mathrm{7}} \\ $$$$\Rightarrow{x}=\sqrt{\mathrm{7}}\approx\mathrm{2}.\mathrm{64575} \\ $$

Answered by mr W last updated on 09/Apr/24

Commented by mr W last updated on 09/Apr/24

HC=(√(12^2 +8^2 −12×8))=4(√7)  (x/y)×(6/6)×((12)/4)=1  ⇒(x/y)=(1/3) ⇒(x/(x+y))=(1/(1+3))=(1/4)  ⇒x=((HC)/4)=(√7)

$${HC}=\sqrt{\mathrm{12}^{\mathrm{2}} +\mathrm{8}^{\mathrm{2}} −\mathrm{12}×\mathrm{8}}=\mathrm{4}\sqrt{\mathrm{7}} \\ $$$$\frac{{x}}{{y}}×\frac{\mathrm{6}}{\mathrm{6}}×\frac{\mathrm{12}}{\mathrm{4}}=\mathrm{1} \\ $$$$\Rightarrow\frac{{x}}{{y}}=\frac{\mathrm{1}}{\mathrm{3}}\:\Rightarrow\frac{{x}}{{x}+{y}}=\frac{\mathrm{1}}{\mathrm{1}+\mathrm{3}}=\frac{\mathrm{1}}{\mathrm{4}} \\ $$$$\Rightarrow{x}=\frac{{HC}}{\mathrm{4}}=\sqrt{\mathrm{7}} \\ $$

Answered by cortano21 last updated on 10/Apr/24

Commented by mr W last updated on 10/Apr/24

FK=((FD)/2) is not obvious, must be  proved at first.

$${FK}=\frac{{FD}}{\mathrm{2}}\:{is}\:{not}\:{obvious},\:{must}\:{be} \\ $$$${proved}\:{at}\:{first}. \\ $$

Commented by A5T last updated on 10/Apr/24

(4/8)×((12)/6)×((DK)/(KF))=1⇒DK=KF⇒FK=((FD)/2)

$$\frac{\mathrm{4}}{\mathrm{8}}×\frac{\mathrm{12}}{\mathrm{6}}×\frac{{DK}}{{KF}}=\mathrm{1}\Rightarrow{DK}={KF}\Rightarrow{FK}=\frac{{FD}}{\mathrm{2}} \\ $$

Terms of Service

Privacy Policy

Contact: info@tinkutara.com